Jump to content

MPMin

Senior Members
  • Posts

    262
  • Joined

  • Last visited

Everything posted by MPMin

  1. Yes but ionised plasma is still a propellant which will eventually be used up, my proposal doesn’t require a propellant and could potentially have an endless supply of thrust from solar power.
  2. I thought you said it was irrelevant? Can you even justify this against what I said? Perhaps you’ve missed the point of my concept, mine has no moving parts either. Are you still talking about the same thing? Based on what exactly? Are you able to give this statement some tangible reasoning beyond supposition? I’m not sure this statement supports your argument as ‘gazillion’ isn’t a real number. This is the correct context to use the word a bogus. I haven’t crunched the numbers yet, have you? I believe this is your low estimate, but I don’t believe it applies to every possible application in space. Are you saying that all satellites are over 10 tons in mass because the space shuttle has a greater mass? I think the amount of energy it takes to get a satellite would put pressure on engineers to keep satellites as light as possible. Actually searching for the answer on google reveals that a lot of satellites weigh a lot less then 10 tons.
  3. If the force on the wire is irrelevant, what’s going to move the craft? If an intermittent force is collectively greater than a really weak continuous force of you know like a laser then it is better. And the multiplication factor is not fake, what the total product of force ends up being is yet to be determined but certainly not fake. If the system can gather endless energy from the sun to power itself and produce thrust over time, perhaps you could explain why ‘efficiency’ would be a failure point? The wind blowing a rock on the ground is hardly a relative comparison to moving a craft in space and without a complete design how can you assume the mass of the craft?
  4. I’m pretty sure efficiency, or lack there of, doesn’t prevent a force from moving something in space. I understand each cycle would only produce a tiny amount of force but keep in mind that that’s only a tiny amount of force per cycle. Im not saying the wires should be 0.1m apart; but if they were, and light travels 300 million meters per second, and each cycle occurs in the time it takes one pulse to reach the other wire, even if each pulse only produces a tiny amount of force, that force would be multiplied 3 billion (3x10^9) times per second, in one week of gathering momentum that tiny force will be multiplied by 1.8x10^15.
  5. I haven’t been given a model, I’ve been the one proposing a model this whole time. The reference you have kindly provided does not refer to my ‘rig’, your reference talks about ejecting mass for propulsion but thank you all the same. Weak or not it would seem that my idea at least has merit and the potential to work as a reactionless propulsion system. Requiring only solar power to generate propulsion (if it actually works) efficiency and power aren’t so much of a concern because even a weak propulsion system that can continually refuel itself from the sun will have the ability to gather momentum over time.
  6. At t Yes, and as I already described, the -p momentum is counteracted by p at time t=0, p then interacts with the system again when it arrives at B and -p does not work means the system will generate momentum to the left
  7. Yes I wrong to say the emp doesn’t carry any momentum or that it was conditional. What I was trying to say was that the momentum in -p ‘effectively’ becomes 0 because it doesn’t interact with the system or should I say it doesn’t transfer its momentum into the system. Are you saying it would work now?
  8. Yes! and that’s the point isn’t it? Using your analogy, the wire acts as a back board in left direction along the x axis while the other tennis ball rock or emp just goes out the back end affecting nothing in the right direction hence giving a net force to the left. If it doesn’t have the ability to transfer it’s momentum then it doesn’t transfer it! The emp tennis ball rock are thrown simultaneously in opposite directions along the x axis, (we can ignore all the other y directions as Ghidein pointed out ) The two objects being thrown in opposite directions cancel each other out, however in the left direction theres a collision from one of the objects being thrown affecting the movement of the craft to the left while nothing happens to the right - the net effect to the right is 0 because nothing happens in that direction. To refer back to Ghideons drawing, when p and - p leave the source wire their effect on the source wire are cancelled out by each other, however, p will collide with B causing a movement to the left while -p does nothing, the net effect of -p is 0 because it doesn’t transfer its momentum to the craft but p does transfer it’s momentum to the craft moving it to the left.
  9. Thank you Ghideon for your very detailed analysis, your drawing will now help me explain my theory even better. I completely agree with everything you said except for one crucial thing, I do not agree that a electromagnetic field pulse (emp) has any momentum ‘unless it interacts with a wire carrying a current’ only then does the emp have momentum at the vicinity of interaction. The reason I believe an emp can have no momentum is because an emp can pass through a non-conductive material having no effect on the non-conductive material and nor does the non-conductive material have any effect on the emp passing through it, hence no apparent momentum in the emp. The momentum of an emp seems to be conditional. The condition to induce momentum from an emp is to obstruct the emp with a wire carrying a current, the wire must be carrying a current at the time of interaction. A wire that’s not carrying a current in the path of an emp, the emp will just induce a current in the wire instead of movement. However, if the wire is carrying a current when the emp interacts with the wire, I believe the current in the wire causes a resistance to the emp which causes the momentum. As the emp is circular, I do not believe this phenomenon will affect the opposite side of the emp (at -p in Ghideon’s drawing) as that part of the emp is not in the vicinity of the current carrying wire (at B and p with reference to Ghideon’s drawing), hence, for all intensive purposes, with reference to Gideon’s drawing, -p always remains 0 (or effectively becomes 0 the further away it is from B and P) as it does not interact with any other magnetic field or current carrying wire on that side, hence -p has no momentum.
  10. Im obviously missing something, Ghideon says the complete rig moves to the left?
  11. Yes lets call the initiating wire (the first wire to fire being the one on the right) A and the other B. Ive thought about this and yes, conventionally, when an emp interacts with a wire it induces a current, but what if that wire is already carrying a current? I believe if that wire is already carrying a current then the emp will induce a force instead. To clarify, If the emp had the potential to induce a current greater than the receiving wire was already carrying, the emp would induce a force to the equivalent magnitude of the current in the receiving wire and the excess emp potential would induce a current in the wire. In other words, if the receiving wire was not carrying a current, 100% of the emp’s power would induce a current in the receiving wire. If the emp had the potential to induce half the current being carried then half the emp’s potential would be converted to Newton’s. Ill try to describe this better latter but essentially the current carried in the receiving wire will act as resistance causing the emp to produce Newtons instead of current in the receiving wire.
  12. I was trying to describe the Newton’s of force per second in the same direction or the net Newton’s of force per second. The system might only work on a single cycle as per my first animation or it might even work with the additional cycle which would increase the number of effective impulses hence Newton’s per second. I have read your analysis but I don’t fully understand what you are saying. It seems you are saying the system will move in the desired direction. As you haven’t said it will work in a ‘yes or no’ answer I don’t want to assume your conclusion. The only question I have relating to your analysis is: are you assuming the circular nature of the emp has some kind of structural integrity? The reason I as this is because you seem to say that the force on p has an effect on -p or visa versa (either way, it seems you are saying they are interrelated which suggests to me the circle has an integral structure or that the emp circle emanating away from wire A applies a continual force on wire A even though it’s emanating away from wire A? Another way of thinking of the emp from wire A is imagine a sound wave emanating from wire A, the effect the sound wave has at wire B has no bearing on whats happing on the other side of the sound wave or emp. I hope this makes sense. im just looking for a conclusive answer with sound reasoning.
  13. Yes, I was referring to N/s. Do you think this system could work?
  14. Here is an updated animation with green arrows depicting the direction of the magnetic fields and also I’ve added the supplementary cycle which could possibly increase the force per time output if it worked. 6BAEBA6B-75DC-4594-A13D-21B2402202E8.mp4
  15. The two wires are on seperate circuits and are not connected to each other. The system would require the two circuits to be in tune to ‘fire’ at exactly the right time.
  16. Yes, that is what i am proposing. Just in case it does work, there is also the possibility of utilising the second magnetic pulse as it arrives back at the source wire, the source wire could then have the current pulsed in the other direction so that it is attracted to the on coming magnetic pulse thus producing a force at the originals source wire in the same direction. However, There’s no point exploring this if the above cycle wont work.
  17. Currents are as shown in the wires. the animation above depicts one basic cycle, at the end of the cycle the process repeats to generate more force on the wire. This process is happening at an extremely fast rate enabling many pulses of force to occur per second so that even if the force per cycle was very small, the cumulative effect could be a large force. My theory relies on the fact that the magnetic pulse will only cause a force on the wire as shown. The wire experiencing the force, while being attached to the craft will push the craft and the other wire will be unaffected as its not carrying a current in that moment.
  18. Do you agree with this animation then? please forgive my drawing ability 6929FEE2-7072-4769-80C1-9E817A79C51D.mp4
  19. 1. For my theory to work, firstly relies on the fact that two wires in parallel, in reasonably close proximity to each other, carrying a continuous current will exert a force on each other. https://www.khanacademy.org/science/physics/magnetic-forces-and-magnetic-fields/magnetic-field-current-carrying-wire/v/magnetism-7 2. The next principle my theory relies upon is that magnetic fields travel and it needs to be able to travel to become an emp. https://physics.stackexchange.com/questions/299947/does-electric-and-magnetic-fields-travel-or-they-just-appear-in-space https://en.m.wikipedia.org/wiki/Electromagnetic_pulse 3. The next principle relies on an emp being able push or pull a wire already carrying a current. Unfortunately i cant find any references exploring this thus I’m assuming based on the above; that if a continuous magnetic field (created by a continuous current) will exert a force on a wire then an intermittent magnetic field or emp will as well?
  20. Someone here said that if any of the wires are attached to the craft then its considered internal, are you using ‘internal’ in the same context? Perhaps if i explain my theory in seperate aspects you could tell me which part of my theory is wrong? Is it ok if i reconstruct my theory in a ‘if then’ sequence? If the system requires an energy input and loses energy from the system, can it be said in no uncertain terms that it’s violating newtons law of conservation?
  21. Its hard for me to accept what you are saying when what you are saying is unclear to me. You say I haven’t understood but the only thing I’m having trouble understanding is how you are drawing your conclusions. When you make statements that are out of context i have trouble understanding your reasoning. I know what you all have been saying but you don’t seem to understand what I’ve been saying so I’m struggling to accept your reasoning on this matter. A lot of what you are saying is really out of context, for example, you say with distance the field will drop to zero, in general terms yes but it really depends how small a percentage of the original field strength you want to regard as negligible (rather than actual ‘zero’ ) and i am well aware that field strength is inversely proportional to the distance squared but again this comment is out of context as I’m not talking about huge distances between the wires in my proposal. You also say that such a field can only exist around the wire creating it as if to suggest my proposal wont produce a magnetic field but if you look at my drawing that’s exactly what I’ve drawn - a magnetic field around the wire, so when i read all these out of context comments, how can you expect me to just accept what you are saying. Further more you then you go on to say that of course that magnetic field will produce a force on the wire in the diagram (after you more or less said it wouldn’t produce a magnetic field) on the wire. We’ve gone through four or five pages of you telling me i cant violate newtons laws ( Id like to add a note here to Ghideon while you are laughing at me with my finger in my guass gun I’m laughing at you coming across as being religious defending your god newton and his commandments I don’t mean that in a bad way I did see the funny side of what you said) when as it turns out now you add a disclaimer about the conservation of energy saying they don’t apply but oh they do if you measure all the immeasurables. I do appreciate all the time you’ve taken on this with me and I’d even more appreciate it if you all were a little more objective about this and really try understand what I’m saying first before just jumping to conclusions. I’m sorry i frightened you all with suggesting its a reaction less drive maybe it’s not reactionless, does that make you all feel better? Ive come to you all as the experts in your field and I’m looking for certain answers, not suppositions. If I’m wrong then I’m wrong that’s fine but please show me in no uncertain terms that I’m wrong.
  22. This is an inaccurate representation of my idea and still makes me wonder if I’ve explained myself adequately. You are proposing that the magnetic field is trying to push itself off of its mounting where as I’m saying its generating a ‘casually detached’ magnetic field to push or pull on. i understand that generating a magnetic pulse will also produce other forms of energy which I understand to mean that 100% of the energy used to produce the magnetic field will not be 100% efficient, i never suggested it would, Ive said all along the system will require an energy input. If this is used in space perhaps the low temperature will cause the wire to have no resistance but that’s the best case scenario as some of the energy will be lost in other forms. Its still not clear to me if the law of conservation of energy applies as energy can be lost from the system and on that, energy can be added to the system as well. At this stage the defining question is: will the situation above produce a physical force on the wire as shown by the red arrow and if not, why not in terms of the magnetic field interactions?
×
×
  • Create New...

Important Information

We have placed cookies on your device to help make this website better. You can adjust your cookie settings, otherwise we'll assume you're okay to continue.